Additional TB Health Assessment Questions chapters

Réussis tes devoirs et examens dès maintenant avec Quizwiz!

The nurse notices that a patient's palpebral fissures are unequal. On examination, the nurse may find that damage has occurred to which cranial nerve (CN)? a. III b. V c. VII d. VIII

ANS: C Facial muscles are mediated by CN VII; asymmetry of palpebral fissures may be attributable to damage to CN VII (Bell's palsy).

During an otoscopic examination, the nurse notices an area of black and white dots on the tympanic membrane and the ear canal wall. What does this finding suggest? a. Malignancy b. Viral infection c. Blood in the middle ear d. Yeast or fungal infection

ANS: D A colony of black or white dots on the drum or canal wall suggests a yeast or fungal infection (otomycosis).

During an interview, the patient states he has the sensation that "everything around him is spinning." The nurse recognizes that the portion of the ear responsible for this sensation is the: a. Cochlea b. CN VIII c. Organ of Corti d. Labyrinth

ANS: D If the labyrinth ever becomes inflamed, it feeds the wrong information to the brain, creating a staggering gait and a strong spinning and whirling sensation called vertigo.

A 50-year-old woman is in the clinic for weakness in her left arm and leg that she has noticed for the past week. The nurse should perform which type of neurologic examination? a. Glasgow Coma Scale b. Neurologic recheck examination c. Screening neurologic examination d. Complete neurologic examination

ANS: D The nurse should perform a complete neurologic examination on an individual who has neurologic concerns (e.g., headache, weakness, loss of coordination) or who is showing signs of neurologic dysfunction. The Glasgow Coma Scale is used to define a persons level of consciousness. The neurologic recheck examination is appropriate for those who are demonstrating neurologic deficits. The screening neurologic examination is performed on seemingly well individuals who have no significant subjective findings from the health history.

The nurse recognizes that a patient has a normal pupillary light reflex when: a. The eyes converge to focus on the light. b. Light is reflected at the same spot in both eyes. c. The eye focuses the image in the centre of the pupil. d. Constriction of both pupils occurs in response to bright light.

ANS: D The pupillary light reflex is the normal constriction of the pupils when bright light shines on the retina. The other responses are not correct.

The nurse is taking the history of a patient who may have a perforated eardrum. What would be an important question in this situation? a. "Do you ever notice ringing or crackling in your ears?" b. "When was the last time you had your hearing checked?" c. "Have you ever been told that you have any type of hearing loss?" d. "Do you have any ear pain or discharge, and if so, when did they occur?"

ANS: D Typically with perforation, ear pain occurs first, stopping with a popping sensation, and then drainage occurs.

In an individual with otitis externa, which of these signs would the nurse expect to find on assessment? a. Rhinorrhea b. Periorbital edema c. Pain over the maxillary sinuses d. Enlarged superficial cervical nodes

ANS: D The lymphatic drainage of the external ear flows to the parotid, mastoid, and superficial cervical nodes. The signs are severe swelling of the canal, inflammation, and tenderness. Rhinorrhea, periorbital edema, and pain over the maxillary sinuses do not occur with otitis externa.

During the taking of the health history of a 78-year-old man, his wife states that he occasionally has problems with short-term memory loss and confusion: He cant even remember how to button his shirt. When assessing his sensory system, which action by the nurse is most appropriate? a. The nurse would not test the sensory system as part of the examination because the results would not be valid. b. The nurse would perform the tests, knowing that mental status does not affect sensory ability. c. The nurse would proceed with an explanation of each test, making certain that the wife understands. d. Before testing, the nurse would assess the patients mental status and ability to follow directions.

ANS: D The nurse should ensure the validity of the sensory system testing by making certain that the patient is alert, cooperative, comfortable, and has an adequate attention span. Otherwise, the nurse may obtain misleading and invalid results.

During an assessment of a 32-year-old patient with a recent head injury, the nurse notices that the patient responds to pain by extending, adducting, and internally rotating his arms. His palms pronate, and his lower extremities extend with plantar flexion. Which statement concerning these findings is most accurate? This patients response: a. Indicates a lesion of the cerebral cortex. b. Indicates a completely nonfunctional brainstem. c. Is normal and will go away in 24 to 48 hours. d. Is a very ominous sign and may indicate brainstem injury.

ANS: D These findings are all indicative of decerebrate rigidity, which is a very ominous condition and may indicate a brainstem injury.

The nurse notices that a patient's submental lymph nodes are enlarged. To identify the cause of the enlargement of the patient's nodes, the nurse assesses the: a. Infraclavicular area. b. Supraclavicular area. c. Area distal to the enlarged node. d. Area proximal to the enlarged node.

ANS: D When nodes are abnormal, the nurse should check the area into which they drain for the source of the problem. The area proximal (upstream) to the location of the abnormal node should be explored.

While performing the otoscopic examination of a 3-year-old boy who has been pulling on his left ear, the nurse finds that his left tympanic membrane is bright red and that the light reflex is not visible. The nurse interprets these findings to indicate: a. Fungal infection. b. Acute otitis media. c. Perforation of the eardrum. d. Cholesteatoma.

ANS: B Absent or distorted light reflex and a bright red colour of the eardrum are indicative of acute otitis media. (See Table 16-5 and 16-6.)

When assessing a male child for colour deficiency the nurse will: a. Check colour vision annually until age 18 years. b. Ask the child to identify the colour of his or her clothing. c. Test for colour vision once between ages 4 and 8 years. d. Begin colour vision screening at the child's 2-year checkup.

ANS: C Test boys only once for colour vision between ages 4 and 8 years. Colour vision is not tested in girls because colour deficiency is rare in girls. Testing is performed with the Ishihara test, which is a series of polychromatic cards.

The nurse needs to palpate the temporomandibular joint for crepitation. This joint is located just below the temporal artery and anterior to the: a. Hyoid bone. b. Vagus nerve. c. Tragus. d. Mandible.

ANS: C The temporomandibular joint is just below the temporal artery and anterior to the tragus.

During ocular examinations, the nurse assesses the movement of the extraocular muscles by stimulating: a. Cranial nerves VII and VIII. b. The ciliary body. c. The corneal reflex. d. Cranial nerves III, IV, and VI.

ANS: D Movement of the extraocular muscles is stimulated by three cranial nerves: III, IV, and VI.

When reviewing the structures of the ear with a class of nursing students, the nurse discusses the importance of the function of the eustachian tube in: a. The production of cerumen. b. Remaining open except when swallowing or yawning. c. Allowing the passage of air between the middle and outer ear. d. Helping equalize air pressure on both sides of the tympanic membrane.

ANS: D The eustachian tube allows an equalization of air pressure on each side of the tympanic membrane so that the membrane does not rupture during, for example, altitude changes in an airplane. The tube is normally closed, but it opens with swallowing or yawning.

The nurse is testing the function of CN XI. Which statement best describes the response the nurse should expect if this nerve is intact? The patient: a. Demonstrates the ability to hear normal conversation. b. Sticks out the tongue midline without tremors or deviation. c. Follows an object with his or her eyes without nystagmus or strabismus. d. Moves the head and shoulders against resistance with equal strength.

ANS: D The following normal findings are expected when testing the spinal accessory nerve (CN XI): The patients sternomastoid and trapezius muscles are equal in size; the person can forcibly rotate the head both ways against resistance applied to the side of the chin with equal strength; and the patient can shrug the shoulders against resistance with equal strength on both sides. Checking the patients ability to hear normal conversation checks the function of CN VIII. Having the patient stick out the tongue checks the function of CN XII. Testing the eyes for nystagmus or strabismus is performed to check CNs III, IV, and VI.

The mother of a 2-year-old is concerned because her son has had three ear infections in the past year. What would be an appropriate response by the nurse? a. "It is unusual for a small child to have frequent ear infections unless something else is wrong." b. "We need to check the immune system of your son to determine why he is having so many ear infections." c. "Ear infections are not uncommon in infants and toddlers because they tend to have more cerumen in the external ear." d. "Your son's eustachian tube is shorter and wider than yours because of his age, which allows for infections to develop more easily."

ANS: D The infant's eustachian tube is relatively shorter and wider than the adult's eustachian tube, and its position is more horizontal; consequently, pathogens from the nasopharynx can more easily migrate through to the middle ear. The other responses are not appropriate.

The nurse is reviewing a patients medical record and notes that he is in a coma. Using the Glasgow Coma Scale, which number indicates that the patient is in a coma? a. 6 b. 12 c. 15 d. 24

ANS: A A fully alert, normal person has a score of 15, whereas a score of 7 or less reflects coma on the Glasgow Coma Scale.

In the assessment of a 1-month-old infant, the nurse notices a lack of response to noise or stimulation. The mother reports that in the last week he has been sleeping all of the time, and when he is awake all he does is cry. The nurse hears that the infants cries are very high pitched and shrill. What should be the nurses appropriate response to these findings? a. Refer the infant for further testing. b. Talk with the mother about eating habits. c. Do nothing; these are expected findings for an infant this age. d. Tell the mother to bring the baby back in 1 week for a recheck.

ANS: A A high-pitched, shrill cry or cat-sounding screech occurs with central nervous system damage. Lethargy, hyporeactivity, and hyperirritability, as well as the parents report of significant changes in behavior all warrant referral. The other options are not correct responses.

The nurse is performing an ear examination of an 80-year-old patient. Which of these findings would be considered normal? a. High-tone frequency loss b. Increased elasticity of the pinna c. Thin, translucent membrane d. Shiny, pink tympanic membrane

ANS: A A high-tone frequency hearing loss is apparent for those affected with presbycusis, the hearing loss that occurs with aging. The pinna loses elasticity, causing earlobes to be pendulous. The eardrum may be whiter in colour and more opaque and duller in the older person than in the younger adult.

A 21-year-old patient has a head injury resulting from trauma and is unconscious. There are no other injuries. During the assessment what would the nurse expect to find when testing the patients deep tendon reflexes? a. Reflexes will be normal. b. Reflexes cannot be elicited. c. All reflexes will be diminished but present. d. Some reflexes will be present, depending on the area of injury.

ANS: A A reflex is a defense mechanism of the nervous system. It operates below the level of conscious control and permits a quick reaction to potentially painful or damaging situations.

When performing an otoscopic examination of a 5-year-old child with a history of chronic ear infections, the nurse sees that his right tympanic membrane is amber-yellow in colour and that air bubbles are visible behind the tympanic membrane. The child reports occasional hearing loss and a popping sound with swallowing. The preliminary analysis based on this information is that the child: a. Most likely has serous otitis media. b. Has an acute purulent otitis media. c. Has evidence of a resolving cholesteatoma. d. Is experiencing the early stages of perforation.

ANS: A An amber-yellow colour to the tympanic membrane suggests serum or pus in the middle ear. Air or fluid or bubbles behind the tympanic membrane are often visible. The patient may have feelings of fullness, transient hearing loss, and a popping sound with swallowing. These findings most likely suggest that the child has serous otitis media. The other responses are not correct. (See Table 16-5)

To assess the head control of a 4-month-old infant, the nurse lifts up the infant in a prone position while supporting his chest. The nurse looks for what normal response? The infant: a. Raises the head, and arches the back. b. Extends the arms, and drops down the head. c. Flexes the knees and elbows with the back straight. d. Holds the head at 45 degrees, and keeps the back straight.

ANS: A At 3 months of age, the infant raises the head and arches the back as if in a swan dive. This response is the Landau reflex, which persists until 1 years of age. The other responses are incorrect.

During an examination of a 3-year-old child, the nurse notices a bruit over the left temporal area. The nurse should: a. Continue the examination because a bruit is a normal finding for this age. b. Check for the bruit again in 1 hour. c. Notify the parents that a bruit has been detected in their child. d. Stop the examination, and notify the physician.

ANS: A Bruits are common in the skull in children under 4 or 5 years of age and in children with anemia. They are systolic or continuous and are heard over the temporal area.

When examining a 2-week-old infant the nurse notices that he watches an object but does not follow it with his eyes when moved to different positions. The nurse will: a. Document this as a normal finding. b. Assess the pupillary light reflex for possible blindness. c. Refer the infant to a specialist. d. Continue assessment with the Allen chart.

ANS: A By 2 to 4 weeks, an infant can fixate on an object. By age 1 month, the infant should fixate and follow a bright light or toy.

While discussing the history of a 6-month-old infant, the mother tells the nurse that she took a significant amount of aspirin while she was pregnant. What question would the nurse want to include during history taking? a. "Does your baby seem to startle with loud noises?" b. "Has your baby had any surgeries on her ears?" c. "Have you noticed any drainage from her ears?" d. "How many ear infections has your baby had since birth?"

ANS: A Children at risk for a hearing deficit include those exposed in utero to a variety of conditions, such as maternal rubella or to maternal ototoxic drugs.

The nurse is performing an assessment on a 7-year-old child who has symptoms of chronic watery eyes, sneezing, and clear nasal drainage. The nurse notices the presence of a transverse line across the bridge of the nose, dark blue shadows below the eyes, and a double crease on the lower eyelids. These findings are characteristic of: a. Allergies. b. Sinus infection. c. Nasal congestion. d. Upper respiratory infection.

ANS: A Chronic allergies often develop chronic facial characteristics and include blue shadows below the eyes, a double or single crease on the lower eyelids, open-mouth breathing, and a transverse line on the nose.

The nurse is preparing to perform an otoscopic examination on a 2-year-old child. Which one of these reflects the correct procedure? a. Pulling the pinna down b. Pulling the pinna up and back c. Slightly tilting the child's head toward the examiner d. Instructing the child to touch his chin to his chest

ANS: A For an otoscopic examination on an infant or on a child younger than 3 years of age, the pinna is pulled down. The other responses are not part of the correct procedure.

During a well-baby checkup, a mother is concerned because her 2-month-old infant cannot hold her head up when she is pulled to a sitting position. Which response by the nurse is appropriate? a. "Head control is usually achieved by 4 months of age." b. "You shouldn't be trying to pull your baby up like that until she is older." c. "Head control should be achieved by this time." d. "This inability indicates possible nerve damage to the neck muscles."

ANS: A Head control is achieved by 4 months when the baby can hold the head erect and steady when pulled to a vertical position. The other responses are not appropriate.

In a person with an upper motor neuron lesion such as a cerebrovascular accident, which of these physical assessment findings should the nurse expect? a. Hyperreflexia b. Fasciculations c. Loss of muscle tone and flaccidity d. Atrophy and wasting of the muscles

ANS: A Hyperreflexia, diminished or absent superficial reflexes, and increased muscle tone or spasticity can be expected with upper motor neuron lesions. The other options reflect a lesion of lower motor neurons.

A 68-year-old woman is in the eye clinic for a checkup. She tells the nurse that she has been having trouble reading the paper, sewing, and even seeing the faces of her grandchildren. On examination, the nurse notes that she has some loss of central vision but her peripheral vision is normal. These findings suggest that she may have: a. Macular degeneration. b. Vision that is normal for someone her age. c. The beginning stages of cataract formation. d. Increased intraocular pressure or glaucoma.

ANS: A Macular degeneration is the most common cause of blindness. It is characterized by the loss of central vision. Cataracts would show lens opacity. Chronic open-angle glaucoma, the most common type of glaucoma, involves a gradual loss of peripheral vision. These findings are not consistent with vision that is considered normal at any age.

A patient's vision is recorded as 20/80 in each eye. The nurse interprets this finding to mean that the patient has: a. Impaired vision. b. Exophthalmos. c. Normal vision. d. Presbyopia.

ANS: A Normal visual acuity is 20/20 in each eye; the larger the denominator, the poorer is the vision.

To test for gross motor skill and coordination of a 6-year-old child, which of these techniques would be appropriate? Ask the child to: a. Hop on one foot. b. Stand on his head. c. Touch his finger to his nose. d. Make funny faces at the nurse.

ANS: A Normally, a child can hop on one foot and can balance on one foot for approximately 5 seconds by 4 years of age and can balance on one foot for 8 to 10 seconds at 5 years of age. Children enjoy performing these tests. Failure to hop after 5 years of age indicates incoordination of gross motor skills. Asking the child to touch his or her finger to the nose checks fine motor coordination; and asking the child to make funny faces tests CN VII. Asking a child to stand on his or her head is not appropriate.

A 31-year-old patient tells the nurse that he has noticed a progressive loss in his hearing. He says that it does seem to help when people speak louder or if he turns up the volume of the television or radio. The most likely cause of his hearing loss is: a. Otosclerosis. b. Presbycusis. c. Trauma to the bones. d. Frequent ear infections.

ANS: A Otosclerosis is a common cause of conductive hearing loss in young adults between ages 20 and 40 years. Presbycusis is a type of hearing loss that occurs with aging. Trauma and frequent ear infections are not a likely cause of his hearing loss.

While obtaining a health history of a 3-month-old infant from the mother, the nurse asks about the infants ability to suck and grasp the mothers finger. What is the nurse assessing? a. Reflexes b. Intelligence c. CNs d. Cerebral cortex function

ANS: A Questions regarding reflexes include such questions as, What have you noticed about the infants behavior, Are the infants sucking and swallowing seem coordinated, and Does the infant grasp your finger? The other responses are incorrect.

When performing the corneal light reflex assessment, the nurse notes that the light is reflected at the 2 o'clock position in each eye. The nurse should: a. Consider this a normal finding. b. Refer the individual for further evaluation. c. Document this finding as an asymmetrical light reflex. d. Perform the confrontation test to validate the findings.

ANS: A Reflection of the light on the corneas should be in exactly the same spot on each eye, or symmetrical. If asymmetry is noted, then the nurse should administer the cover test.

During the assessment of an 80-year-old patient, the nurse notices that his hands show tremors when he reaches for something and his head is always nodding. No associated rigidity is observed with movement. Which of these statements is most accurate? a. These findings are normal, resulting from aging. b. These findings could be related to hyperthyroidism. c. These findings are the result of Parkinson disease. d. This patient should be evaluated for a cerebellar lesion.

ANS: A Senile tremors occasionally occur. These benign tremors include an intention tremor of the hands, head nodding (as if saying yes or no), and tongue protrusion. Tremors associated with Parkinson disease include rigidity, slowness, and a weakness of voluntary movement. The other responses are incorrect.

During an ophthalmoscopic examination of the eye, the nurse notices areas of exudate that appear similar to cotton wool or fluffy clouds. The nurse recognizes that the patient may have: a. Diabetes. b. Hyperthyroidism. c. Glaucoma. d. Hypotension.

ANS: A Soft exudates or cotton wool areas that appear similar to fluffy grey-white cumulus clouds occur with diabetes, hypertension, subacute bacterial endocarditis, lupus, and papilledema of any cause. These exudates are not found with hyperthyroidism, glaucoma, or hypotension.

The nurse is testing the deep tendon reflexes of a 30-year-old woman who is in the clinic for an annual physical examination. When striking the Achilles heel and quadriceps muscle, the nurse is unable to elicit a reflex. The nurses next response should be to: a. Ask the patient to lock her fingers and pull. b. Complete the examination, and then test these reflexes again. c. Refer the patient to a specialist for further testing. d. Document these reflexes as 0 on a scale of 0 to 4+.

ANS: A Sometimes the reflex response fails to appear. Documenting the reflexes as absent is inappropriate this soon in the examination. The nurse should try to further encourage relaxation, varying the persons position or increasing the strength of the blow. Reinforcement is another technique to relax the muscles and enhance the response. The person should be asked to perform an isometric exercise in a muscle group somewhat away from the one being tested. For example, to enhance a patellar reflex, the person should be asked to lock the fingers together and pull.

The nurse suspects that a patient has hyperthyroidism, and the laboratory data indicate that the patient's T4 and T3 levels are elevated. During assessment, the nurse will likely find the patient has: a. Tachycardia. b. Constipation. c. Rapid dyspnea. d. Atrophied nodular thyroid gland.

ANS: A T4 and T3 are thyroid hormones that stimulate the rate of cellular metabolism, resulting in tachycardia. With an enlarged thyroid gland as in hyperthyroidism, the nurse might expect to find diffuse enlargement (goitre) or a nodular lump but not an atrophied gland. Dyspnea and constipation are not findings associated with hyperthyroidism.

The nurse knows that determining whether a person is oriented to his or her surroundings will test the functioning of which structure(s)? a. Cerebrum b. Cerebellum c. CNs d. Medulla oblongata

ANS: A The cerebral cortex is responsible for thought, memory, reasoning, sensation, and voluntary movement. The other structures are not responsible for a persons level of consciousness.

The nurse is performing an external eye examination. Which statement regarding the outer layer of the eye is true? a. The outer layer of the eye is very sensitive to touch. b. The outer layer of the eye is darkly pigmented to prevent light from reflecting internally. c. The trigeminal nerve (cranial nerve V) and the trochlear nerve (cranial nerve IV) are stimulated when the outer surface of the eye is stimulated. d. The visual receptive layer of the eye in which light waves are changed into nerve impulses is located in the outer layer of the eye.

ANS: A The cornea and the sclera make up the outer layer of the eye. The cornea is very sensitive to touch. The middle layer, the choroid, has dark pigmentation to prevent light from reflecting internally. The trigeminal nerve (cranial nerve V) and the facial nerve (cranial nerve VII) are stimulated when the outer surface of the eye is stimulated. The retina, in the inner layer of the eye, is where light waves are changed into nerve impulses.

The nurse is administering ear drops to a 68-year-old patient. The nurse will pull the area of the ear consisting of movable cartilage and skin, known as the ___________, upward and backward to open the ear canal. a. Auricle b. Concha c. Outer meatus d. Mastoid process

ANS: A The external ear is called the auricle or pinna and consists of movable cartilage and skin.

The wife of a 65-year-old man tells the nurse that she is concerned because she has noticed a change in her husbands personality and ability to understand. He also cries very easily and becomes angry. The nurse recalls that the cerebral lobe responsible for these behaviors is the __________ lobe. a. Frontal b. Parietal c. Occipital d. Temporal

ANS: A The frontal lobe has areas responsible for personality, behavior, emotions, and intellectual function. The parietal lobe has areas responsible for sensation; the occipital lobe is responsible for visual reception; and the temporal lobe is responsible for hearing, taste, and smell.

When testing a patient's visual accommodation the nurse notes a normal finding when the patient demonstrates: a. Pupillary constriction when looking at a near object. b. Pupillary dilation when looking at a far object. c. Changes in peripheral vision in response to light. d. Involuntary blinking in the presence of bright light.

ANS: A The muscle fibres of the iris contract the pupil in bright light and accommodate for near vision, which also results in pupil constriction. The other responses are not correct.

The nurse is assessing a patient who may have hearing loss. Which of these statements concerning air conduction is true? a. Air conduction is the normal pathway for hearing. b. Vibrations of the bones in the skull cause air conduction. c. Amplitude of sound determines the pitch that is heard. d. Loss of air conduction is called a conductive hearing loss.

ANS: A The normal pathway of hearing is air conduction, which starts when sound waves produce vibrations on the tympanic membrane. Conductive hearing loss results from a mechanical dysfunction of the external or middle ear. The other statements are not true concerning air conduction.

The nurse is examining a patient's retina with an ophthalmoscope and notes the normal finding of: a. An optic disc that is a yellow-orange colour. b. Optic disc margins that are blurred around the edges. c. The presence of pigmented crescents in the macular area. d. The presence of the macula located on the nasal side of the retina.

ANS: A The optic disc is located on the nasal side of the retina. Its colour is a creamy yellow-orange to a pink, and the edges are distinct and sharply demarcated, not blurred. A pigmented crescent is black and is caused by the accumulation of pigment in the choroid.

When examining the patient's eyes, the nurse notices that his eyelid margins approximate completely when closed. The nurse will: a. Document this as a normal finding. b. Evaluate the extraocular muscles. c. Refer the patient for problems with tearing. d. Assess for increased intraocular pressure.

ANS: A The palpebral fissure is the elliptical open space between the eyelids, and, when closed, the lid margins approximate completely, which is a normal finding.

During an assessment of a 62-year-old man, the nurse notices the patient has a stooped posture, shuffling walk with short steps, flat facial expression, and pill-rolling finger movements. These findings would be consistent with: a. Parkinsonism. b. Cerebral palsy. c. Cerebellar ataxia. d. Muscular dystrophy.

ANS: A The stooped posture, shuffling walk, short steps, flat facial expression, and pill-rolling finger movements are all found in parkinsonism.

A patient's laboratory data reveal an elevated thyroxine (T4) level. The nurse would proceed with an examination of the _____ gland. a. Thyroid b. Parotid c. Adrenal d. Parathyroid

ANS: A The thyroid gland is a highly vascular endocrine gland that secretes T4 and triiodothyronine (T3). The other glands do not secrete T4.

Which of these tests would the nurse use to check the motor coordination of an 11-month-old infant? a. Denver II b. Stereognosis c. Deep tendon reflexes d. Rapid alternating movements

ANS: A To screen gross and fine motor coordination, the nurse should use the Denver II with its age-specific developmental milestones. Stereognosis tests a persons ability to recognize objects by feeling them and is not appropriate for an 11-month-old infant. Testing the deep tendon reflexes is not appropriate for checking motor coordination. Testing rapid alternating movements is appropriate for testing coordination in adults.

During the taking of the health history, a patient tells the nurse that it feels like the room is spinning around me. The nurse would document this finding as: a. Vertigo. b. Syncope. c. Dizziness. d. Seizure activity.

ANS: A True vertigo is rotational spinning caused by a neurologic dysfunction or a problem in the vestibular apparatus or the vestibular nuclei in the brainstem. Syncope is a sudden loss of strength or a temporary loss of consciousness. Dizziness is a lightheaded, swimming sensation. Seizure activity is characterized by altered or loss of consciousness, involuntary muscle movements, and sensory disturbances.

During an examination, the nurse knows that the best way to palpate the lymph nodes in the neck is by: a. Using gentle pressure and palpating with both hands to compare the two sides. b. Using strong pressure and palpating with both hands to compare the two sides. c. Gently pinching each node between one's thumb and forefinger and then moving down the neck muscle. d. Using the index and middle fingers and gently palpating by applying pressure in a rotating pattern.

ANS: A Using gentle pressure is recommended because strong pressure can push the nodes into the neck muscles. Palpating with both hands to compare the two sides symmetrically is usually most efficient.

A 59-year-old patient has a herniated intervertebral disk. Which of the following findings should the nurse expect to see on physical assessment of this individual? a. Hyporeflexia b. Increased muscle tone c. Positive Babinski sign d. Presence of pathologic reflexes

ANS: A With a herniated intervertebral disk or lower motor neuron lesion, loss of tone, flaccidity, atrophy, fasciculations, and hyporeflexia or areflexia are demonstrated. No Babinski sign or pathologic reflexes would be observed. The other options reflect a lesion of upper motor neurons.

The nurse assesses the hearing of a 7-month-old by clapping hands. What is the expected response? The infant: a. Turns his or her head to localize the sound. b. Shows no obvious response to the noise. c. Shows a startle and acoustic blink reflex. d. Stops any movement, and appears to listen for the sound.

ANS: A With a loud sudden noise, the nurse should notice the infant turning the head to localize the sound and to respond to his or her own name. A startle reflex and acoustic blink reflex is expected in newborns; at age 3 to 4 months, the infant stops any movement and appears to listen.

During an examination, the nurse notices that the patient stumbles a little while walking and that when she sits down, she holds on to the sides of the chair. The patient states, "It feels like the room is spinning!" The nurse documents that the patient is experiencing: a. Objective vertigo. b. Subjective vertigo. c. Tinnitus. d. Dizziness.

ANS: A With objective vertigo, the patient feels like the room is spinning; with subjective vertigo, the person feels like he or she is spinning. Tinnitus is a sound that comes from within a person; it can be a ringing, crackling, or buzzing sound. It accompanies some hearing or ear disorders. Dizziness is not the same as true vertigo; the person who is dizzy may feel unsteady and lightheaded.

When examining children affected with Down's syndrome (trisomy 21), the nurse looks for the possible presence of: a. Misshapen ears. b. Long, thin neck. c. Thin tongue sticking out. d. Narrow and raised nasal bridge.

ANS: A With the chromosomal aberration trisomy 21, also known as Down's syndrome, head and face characteristics may include upslanting eyes with inner epicanthal folds (small folds of skin at the inner corners), a flat nasal bridge, a small broad flat nose, a protruding thick tongue, ear dysplasia (misshapen ears), a short broad neck with webbing, and small hands with a single palmar crease. (See Table 14-2.)

The nurse is working at a community health fair to promote eye examinations for populations at higher risk to have vision problems, including: (Select all that apply.) a. Indigenous people. b. People of European descent. c. People with diabetes. d. People of African descent. e. People of French descent. f. People with a family history of glaucoma.

ANS: A, C, D, F Patients with a predisposition to visual deficits include those who wear glasses or contact lenses, have diabetes, are of African descent, or have a strong family history of glaucoma, age-related macular degeneration (AMD), or retinal detachment. Incidence of diabetes is high among Indigenous peoples.

The nurse is providing an educational session to parents in the community on concussions. The nurse shares some of the signs and symptoms to watch for after a head injury which can indicate a concussion and the need to seek medical attention: (Select all that apply.) a. Fatigue b. Calmness c. Photophobia d. Happiness e. Feeling woozy f. Insomnia

ANS: A, C, E Signs and symptoms of a concussion are headache, dizziness/feeling woozy, feeling dazed, "seeing stars," sensitivity to light/photophobia, ringing in ears/tinnitus, tiredness/fatigue, nausea, vomiting, irritability, confusion, and disorientation.

A patient comes into the clinic complaining of pain in her right eye. On examination, the nurse sees a pustule at the lid margin that is painful to touch, red, and swollen. The nurse recognizes that this is a: a. Chalazion. b. Hordeolum (stye). c. Dacryocystitis. d. Blepharitis.

ANS: B A hordeolum, or stye, is a painful, red, and swollen pustule at the lid margin. A chalazion is a nodule protruding on the lid, toward the inside, and is nontender, firm, with discrete swelling. Dacryocystitis is an inflammation of the lacrimal sac. Blepharitis is inflammation of the eyelids (see Table 15-3).

The nurse is performing the diagnostic positions test and notes normal findings with: a. Convergence of the eyes. b. Parallel movement of both eyes. c. Nystagmus in extreme superior gaze. d. Lid lag when moving the eyes from a superior to an inferior position.

ANS: B A normal response for the diagnostic positions test is parallel tracking of the object with both eyes. Eye movement that is not parallel indicates a weakness of an extraocular muscle or dysfunction of the cranial nerve that innervates it.

The nurse suspects that a patient has otitis media. Early signs of otitis media include which of these findings of the tympanic membrane? a. Red and bulging b. Lack of mobility c. Retraction with landmarks clearly visible d. Flat, slightly pulled in at the centre, and moving with insufflation

ANS: B An early sign of otitis media is hypomobility of the tympanic membrane. As pressure increases, the tympanic membrane begins to bulge.

The assessment of a 60-year-old patient has taken longer than anticipated. In testing his pain perception, the nurse decides to complete the test as quickly as possible. When the nurse applies the sharp point of the pin on his arm several times, he is only able to identify these as one very sharp prick. What would be the most accurate explanation for this? a. The patient has hyperesthesia as a result of the aging process. b. This response is most likely the result of the summation effect. c. The nurse was probably not poking hard enough with the pin in the other areas. d. The patient most likely has analgesia in some areas of arm and hyperalgesia in others.

ANS: B At least 2 seconds should be allowed to elapse between each stimulus to avoid summation. With summation, frequent consecutive stimuli are perceived as one strong stimulus. The other responses are incorrect.

A patient reports to the nurse that he has been experiencing excruciating headache pain on one side of his head, especially around his eye, forehead, and cheek that lasts approximately one-half to 2 hours, occurring once or twice each day. The nurse suspects that he is having: a. Hypertension. b. Cluster headaches. c. Tension headaches. d. Migraine headaches.

ANS: B Cluster headaches produce pain around the eye, temple, forehead, and cheek and are unilateral and always on the same side of the head. They are excruciating and occur once or twice per day and last one-half to 2 hours each.

The ability that humans have to perform very skilled movements such as writing is controlled by the: a. Basal ganglia. b. Corticospinal tract. c. Spinothalamic tract. d. Extrapyramidal tract.

ANS: B Corticospinal fibers mediate voluntary movement, particularly very skilled, discrete, and purposeful movements, such as writing. The corticospinal tract, also known as the pyramidal tract, is a newer, higher motor system that humans have that permits very skilled and purposeful movements. The other responses are not related to skilled movements.

The nurse is performing a middle ear assessment on a 15-year-old patient who has had a history of chronic ear infections. When examining the right tympanic membrane, the nurse sees the presence of dense white patches. The tympanic membrane is otherwise unremarkable. It is pearly, with the light reflex at the 5 o'clock position and visible landmarks. The nurse should: a. Refer the patient for the possibility of a fungal infection. b. Know that these are scars caused from frequent ear infections. c. Consider that these findings may represent the presence of blood in the middle ear. d. Be concerned about the ability to hear because of this abnormality on the tympanic membrane.

ANS: B Dense white patches on the tympanic membrane are sequelae of repeated ear infections. They do not necessarily affect hearing.

During an assessment of an infant, the nurse notes that the fontanelles are depressed and sunken. The nurse suspects which condition? a. Rickets b. Dehydration c. Mental retardation d. Increased intracranial pressure

ANS: B Depressed and sunken fontanels occur with dehydration or malnutrition. Mental retardation and rickets have no effect on the fontanels. Increased intracranial pressure would cause tense or bulging and possibly pulsating fontanelles.

The nurse is assessing the neurologic status of a patient who has a late-stage brain tumor. With the reflex hammer, the nurse draws a light stroke up the lateral side of the sole of the foot and inward, across the ball of the foot. In response, the patients toes fan out, and the big toe shows dorsiflexion. The nurse interprets this result as: a. Negative Babinski sign, which is normal for adults. b. Positive Babinski sign, which is abnormal for adults. c. Clonus, which is a hyperactive response. d. Achilles reflex, which is an expected response.

ANS: B Dorsiflexion of the big toe and fanning of all toes is a positive Babinski sign, also called up-going toes. This response occurs with upper motor neuron disease of the corticospinal (or pyramidal) tract and is an abnormal finding for adults.

During an examination, the nurse notices severe nystagmus in both eyes of a patient. Which conclusion by the nurse is correct? Severe nystagmus in both eyes: a. Is a normal occurrence. b. May indicate disease of the cerebellum or brainstem. c. Is a sign that the patient is nervous about the examination. d. Indicates a visual problem, and a referral to an ophthalmologist is indicated.

ANS: B End-point nystagmus at an extreme lateral gaze normally occurs; however, the nurse should carefully assess any other nystagmuses. Severe nystagmus occurs with disease of the vestibular system, cerebellum, or brainstem.

A patient is unable to differentiate between sharp and dull stimulations to both sides of her face. The nurse suspects: a. Bell's palsy. b. Damage to the trigeminal nerve. c. Frostbite with resultant paresthesia to the cheeks. d. Scleroderma.

ANS: B Facial sensations of pain or touch are mediated by cranial nerve V, which is the trigeminal nerve. Bell palsy is associated with cranial nerve VII damage. Frostbite and scleroderma are not associated with this problem.

The nurse is assessing a 16-year-old patient who has suffered head injuries from a recent motor vehicle accident. Which of these statements indicates the most important reason for assessing for any drainage from the ear canal? a. If the drum has ruptured, then purulent drainage will result. b. Bloody or clear watery drainage can indicate a basal skull fracture. c. The auditory canal many be occluded from increased cerumen. d. Foreign bodies from the accident may cause occlusion of the canal.

ANS: B Frank blood or clear watery drainage (cerebrospinal fluid leak) after a trauma suggests a basal skull fracture and warrants immediate referral. Purulent drainage indicates otitis externa or otitis media.

During the health history interview with a 65-year-old male patient of African descent with hypertension, the nurse encourages the patient to have regular eye examinations because of his risk for: a. Cataract. b. Glaucoma. c. Strabismus. d. Proptosis.

ANS: B Health care providers should encourage regular eye examinations, especially for patients with known risk factors. Risk factors for glaucoma include African descent, age greater than 60 years, and hypertension. (See Promoting Health: Screening for Glaucoma).

During a physical education class, a student is hit in the eye with the end of a baseball bat. When examined in the emergency department, the nurse notices the presence of blood in the anterior chamber of the eye indicating the presence of: a. Hypopyon. b. Hyphema. c. Corneal abrasion. d. Pterygium.

ANS: B Hyphema is the term for blood in the anterior chamber and is a serious result of blunt trauma (a fist or a baseball) or spontaneous hemorrhage and may indicate scleral rupture or major intraocular trauma. (See Table 15-7 for descriptions of the other terms.)

A patient in her first trimester of pregnancy is diagnosed with rubella. Which of these statements is correct regarding the significance of this in relation to the infant's hearing? a. Rubella may affect the mother's hearing, but not the infant's. b. Rubella can damage the infant's organ of Corti, which will impair hearing. c. Rubella is only dangerous to the infant in the second trimester of pregnancy. d. Rubella can impair the development of CN VIII and thus affect hearing.

ANS: B If maternal rubella infection occurs during the first trimester, then it can damage the organ of Corti and impair hearing in the child.

During an assessment of a 22-year-old woman who sustained a head injury from an automobile accident 4 hours earlier, the nurse notices the following changes: pupils were equal, but now the right pupil is fully dilated and nonreactive, and the left pupil is 4 mm and reacts to light. What do these findings suggest? a. Injury to the right eye b. Increased intracranial pressure c. Test inaccurately performed d. Normal response after a head injury

ANS: B In a person with a brain injury, a sudden, unilateral, dilated, and nonreactive pupil is ominous. CN III runs parallel to the brainstem. When increasing intracranial pressure pushes down the brainstem (uncal herniation), it puts pressure on CN III, causing pupil dilation. The other responses are incorrect.

The nurse knows that testing kinesthesia is a test of a persons: a. Fine touch. b. Position sense. c. Motor coordination. d. Perception of vibration.

ANS: B Kinesthesia, or position sense, is the persons ability to perceive passive movements of the extremities. The other options are incorrect.

The nurse has just completed a lymph node assessment on a 60-year-old healthy female patient. The nurse knows that most lymph nodes in healthy adults are normally: a. Shotty. b. Nonpalpable. c. Large, firm, and fixed to the tissue. d. Rubbery, discrete, and mobile.

ANS: B Most lymph nodes are nonpalpable in adults. The palpability of lymph nodes decreases with age. Normal nodes feel movable, discrete, soft, and nontender.

A patient says that she has recently noticed a lump in the front of her neck below her "Adam's apple" that seems to be getting bigger. During assessment, the nurse suspects a noncancerous finding as the lump: a. Is singular and firm. b. Consists of multiple nodules. c. Disappears when the patient smiles. d. Is hard and fixed to the surrounding structures.

ANS: B Multiple nodules usually indicate inflammation or a multinodular goitre, rather than a neoplasm. Any rapidly enlarging or firm nodule should be further investigated. Painless, rapidly growing nodules may be cancerous, especially the appearance of a single nodule in a young person. Cancerous nodules tend to be hard and fixed to surrounding structures.

The nurse is performing an eye assessment on an 80-year-old patient and is concerned about finding that the patient has: a. Decreased tear production. b. Unequal pupillary constriction in response to light. c. Arcus senilis around the cornea. d. Loss of the outer hair on the eyebrows attributable to a decrease in hair follicles.

ANS: B Pupils are small in the older adult, and the pupillary light reflex may be slowed, but pupillary constriction should be symmetrical. The assessment findings in the other responses are considered normal in older persons.

A 31-year-old patient tells the nurse that he has noticed pain in his left ear when people speak loudly to him. The nurse knows that this finding: a. Is normal for people of his age. b. Is a characteristic of recruitment. c. May indicate a middle ear infection. d. Indicates that the patient has a cerumen impaction.

ANS: B Recruitment is significant hearing loss occurring when speech is at low intensity, but sound actually becomes painful when the speaker repeats at a louder volume. The other responses are not correct.

In obtaining a health history on a 74-year-old patient, the nurse notes that he drinks alcohol daily and that he has noticed a tremor in his hands that affects his ability to hold things. With this information, what response should the nurse make? a. Does your family know you are drinking every day? b. Does the tremor change when you drink alcohol? c. Well do some tests to see what is causing the tremor. d. You really shouldnt drink so much alcohol; it may be causing your tremor.

ANS: B Senile tremor is relieved by alcohol, although not a recommended treatment. The nurse should assess whether the person is abusing alcohol in an effort to relieve the tremor.

The nurse places a key in the hand of a patient and he identifies it as a penny. What term would the nurse use to describe this finding? a. Extinction b. Astereognosis c. Graphesthesia d. Tactile discrimination

ANS: B Stereognosis is the persons ability to recognize objects by feeling their forms, sizes, and weights. Astereognosis is an inability to identify objects correctly, and it occurs in sensory cortex lesions. Tactile discrimination tests fine touch. Extinction tests the persons ability to feel sensations on both sides of the body at the same point.

A patient has come in for an examination and states, "I have this spot in front of my ear lobe on my cheek that seems to be getting bigger and is tender. What do you think it is?" The nurse notes swelling below the angle of the jaw and suspects that it could be an inflammation of his: a. Thyroid gland. b. Parotid gland. c. Occipital lymph node. d. Submental lymph node.

ANS: B Swelling of the parotid gland is evident below the angle of the jaw and is most visible when the head is extended. Painful inflammation occurs with mumps, and swelling also occurs with abscesses or tumours. Swelling occurs anterior to the lower ear lobe.

During an assessment of the CNs, the nurse finds the following: asymmetry when the patient smiles or frowns, uneven lifting of the eyebrows, sagging of the lower eyelids, and escape of air when the nurse presses against the right puffed cheek. This would indicate dysfunction of which of these CNs? a. Motor component of CN IV b. Motor component of CN VII c. Motor and sensory components of CN XI d. Motor component of CN X and sensory component of CN VII

ANS: B The findings listed reflect a dysfunction of the motor component of the facial nerve (CN VII).

Which statement concerning the areas of the brain is true? a. The cerebellum is the center for speech and emotions. b. The hypothalamus controls body temperature and regulates sleep. c. The basal ganglia are responsible for controlling voluntary movements. d. Motor pathways of the spinal cord and brainstem synapse in the thalamus.

ANS: B The hypothalamus is a vital area with many important functions: body temperature controller, sleep center, anterior and posterior pituitary gland regulator, and coordinator of autonomic nervous system activity and emotional status. The cerebellum controls motor coordination, equilibrium, and balance. The basal ganglia control autonomic movements of the body. The motor pathways of the spinal cord synapse in various areas of the spinal cord, not in the thalamus.

The nurse notices the presence of periorbital edema when performing an eye assessment on a 70-year-old patient. The nurse should: a. Check for the presence of exophthalmos. b. Suspect that the patient has hyperthyroidism. c. Ask the patient if he or she has a history of heart failure. d. Assess for blepharitis, which is often associated with periorbital edema.

ANS: C Periorbital edema occurs with local infections, crying, and systemic conditions, such as heart failure, renal failure, allergy, and hypothyroidism. Periorbital edema is not associated with blepharitis.

The nurse is conducting a visual examination. Which of these statements regarding visual pathways and visual fields is true? a. The right side of the brain interprets the vision for the right eye. b. The image formed on the retina is upside down and reversed from its actual appearance in the outside world. c. Light rays are refracted through the transparent media of the eye before striking the pupil. d. Light impulses are conducted through the optic nerve to the temporal lobes of the brain.

ANS: B The image formed on the retina is upside down and reversed from its actual appearance in the outside world. The light rays are refracted through the transparent media of the eye before striking the retina, and the nerve impulses are conducted through the optic nerve tract to the visual cortex of the occipital lobe of the brain. The left side of the brain interprets vision for the right eye.

A mother of a 1-month-old infant asks the nurse why it takes so long for infants to learn to roll over. The nurse knows that the reason for this is: a. A demyelinating process must be occurring with her infant. b. Myelin is needed to conduct the impulses, and the neurons of a newborn are not yet myelinated. c. The cerebral cortex is not fully developed; therefore, control over motor function gradually occurs. d. The spinal cord is controlling the movement because the cerebellum is not yet fully developed.

ANS: B The infants sensory and motor development proceeds along with the gradual acquisition of myelin, which is needed to conduct most impulses. Very little cortical control exists, and the neurons are not yet myelinated. The other responses are not correct.

The nurse is reviewing age-related changes of the eye for a class. Which of these physiological changes is responsible for presbyopia? a. Degeneration of the cornea b. Loss of lens elasticity c. Decreased adaptation to darkness d. Decreased distance vision abilities

ANS: B The lens loses elasticity and decreases its ability to change shape to accommodate for near vision. This condition is called presbyopia.

When examining a patient after a biopsy of the cervical lymph nodes, to ensure there is no damage to the major neck muscles, the nurse should check the function of cranial nerve: a. V; trigeminal nerve. b. XI; spinal accessory nerve. c. VII; facial nerve. d. VI; abducens nerve.

ANS: B The major neck muscles are the sternomastoid and the trapezius. They are innervated by cranial nerve XI, the spinal accessory. Injury or surgery to the neck region can cause trauma to the nerve.

A patient comes to the clinic complaining of neck and shoulder pain and is unable to turn her head. The nurse suspects damage to cranial nerve ______ and proceeds with the examination by ________________________. a. XI; palpating the anterior and posterior triangles b. XI; asking the patient to shrug her shoulders against resistance c. XII; percussing the sternomastoid and submandibular neck muscles d. XII; assessing for a positive Romberg sign

ANS: B The major neck muscles are the sternomastoid and the trapezius. They are innervated by cranial nerve XI, the spinal accessory. The innervated muscles assist with head rotation and head flexion, movement of the shoulders, and extension and turning of the head.

The two parts of the nervous system are the: a. Motor and sensory. b. Central and peripheral. c. Peripheral and autonomic. d. Hypothalamus and cerebral.

ANS: B The nervous system can be divided into two partscentral and peripheral. The central nervous system includes the brain and spinal cord. The peripheral nervous system includes the 12 pairs of cranial nerves (CNs), the 31 pairs of spinal nerves, and all of their branches.

A mother brings her newborn in for an assessment and asks, "Is there something wrong with my baby? His head seems so big." Which statement is true regarding the relative proportions of the head and trunk of the newborn? a. At birth, the head is one-fifth the total length. b. Head circumference should be greater than chest circumference at birth. c. The head size reaches 90% of its final size when the child is 3 years old. d. When the anterior fontanelle closes at 2 months, the head will be more proportionate to the body.

ANS: B The nurse recognizes that during the fetal period, head growth predominates. Head size is greater than chest circumference at birth, and the head size grows during childhood, reaching 90% of its final size when the child is 6 years of age.

A male patient with a history of AIDS has come in for an examination and he states, "I think that I have the mumps." The nurse would begin by examining the: a. Thyroid gland. b. Parotid gland. c. Cervical lymph nodes. d. Mouth and skin for lesions.

ANS: B The parotid gland may become swollen with the onset of mumps, and parotid enlargement has been found with human immunodeficiency virus (HIV).

A patient's vision is recorded as 20/30 when the Snellen eye chart is used. The nurse interprets these results to indicate that: a. At 30 feet the patient can read the entire chart. b. The patient can read at 20 feet what a person with normal vision can read at 30 feet. c. The patient can read the chart from 20 feet in the left eye and 30 feet in the right eye. d. The patient can read from 30 feet what a person with normal vision can read from 20 feet.

ANS: B The top number indicates the distance the person is standing from the chart; the denominator gives the distance at which a normal eye can see an object.

The physician reports that a patient with a neck tumour has a tracheal shift. The nurse is aware that this means that the patient's trachea is: a. Pulled to the affected side. b. Pushed to the unaffected side. c. Pulled downward. d. Pulled downward in a rhythmic pattern.

ANS: B The trachea is pushed to the unaffected side with an aortic aneurysm, a tumour, unilateral thyroid lobe enlargement, or a pneumothorax. The trachea is pulled to the affected side with large atelectasis, pleural adhesions, or fibrosis. Tracheal tug is a rhythmic downward pull that is synchronous with systole and occurs with aortic arch aneurysm.

When examining the ear with an otoscope, the nurse notes that the tympanic membrane is translucent and pearly. The nurse will: a. Seek assistance from a colleague. b. Document the finding as normal. c. Refer the patient to a specialist. d. Recommend irrigating the ear.

ANS: B The tympanic membrane is a translucent membrane with a pearly grey colour and a prominent cone of light in the anteroinferior quadrant, which is the reflection of the otoscope light. The tympanic membrane is oval and slightly concave, pulled in at its centre by the malleus, which is one of the middle ear ossicles.

During the neurologic assessment of a healthy 35-year-old patient, the nurse asks him to relax his muscles completely. The nurse then moves each extremity through full range of motion. Which of these results would the nurse expect to find? a. Firm, rigid resistance to movement b. Mild, even resistance to movement c. Hypotonic muscles as a result of total relaxation d. Slight pain with some directions of movement

ANS: B Tone is the normal degree of tension (contraction) in voluntarily relaxed muscles. It shows a mild resistance to passive stretching. Normally, the nurse will notice a mild, even resistance to movement. The other responses are not correct.

A 17-year-old student is a swimmer on her high school's swim team. She has had three bouts of otitis externa this season and wants to know what to do to prevent it. The nurse instructs her to: a. Use a cotton-tipped swab to dry the ear canals thoroughly after each swim. b. Use rubbing alcohol or 2% acetic acid eardrops after every swim. c. Irrigate the ears with warm water and a bulb syringe after each swim. d. Rinse the ears with a warmed solution of mineral oil and hydrogen peroxide.

ANS: B With otitis externa (swimmer's ear), swimming causes the external canal to become waterlogged and swell; skinfolds are set up for infection. Otitis externa can be prevented by using rubbing alcohol or 2% acetic acid eardrops after every swim.

A patient comes to the emergency department after a boxing match, and his left eye is swollen almost shut. He has bruises on his face and neck. He says he is worried because he "can't see well" from his left eye. The physician suspects retinal damage. The nurse recognizes that signs of retinal detachment include: a. Loss of central vision. b. Shadow or diminished vision in one quadrant or one-half of the visual field. c. Loss of peripheral vision. d. Sudden loss of pupillary constriction and accommodation.

ANS: B With retinal detachment, the person has shadows or diminished vision in one quadrant or one-half of the visual field. The other responses are not signs of retinal detachment.

In performing a voice test to assess hearing, which of these actions would the nurse perform? a. Shield the lips so that the sound is muffled. b. Whisper a set of random numbers and letters and then ask the patient to repeat them. c. Ask the patient to place his finger in his ear to occlude outside noise. d. Stand approximately 4 feet away to ensure that the patient can really hear at this distance.

ANS: B With the head half a metre (2 feet) from the patient's ear, the examiner exhales and slowly whispers a set of random numbers and letters, such as "5, B, 6." Normally, the patient is asked to repeat each number and letter correctly after hearing the examiner say them.

The nurse is working with new parents to help decrease the incidence of childhood acute otitis media. The nurse provides recommendations which include: (Select all that apply.) a. Propping the bottle for the baby in bed b. Having a smoke-free home c. Avoid letting the baby have a bottle lying flat in bed d. Smoking in vehicle with the child, not in the home e. Encouragement of breastfeeding from birth

ANS: B, C, E The following risk factors predispose children to acute otitis media: absence of breastfeeding in the first 3 months of age, exposure to second-hand tobacco smoke, day care attendance, male sex, pacifier use, low birth weight, low socioeconomic status, and formula feeding in the supine position. Instruct parents to not prop the bottle or have the baby feed from the bottle in bed. Encouraging breastfeeding helps prevent this problem.

A 69-year-old patient has been admitted to an adult psychiatric unit because his wife thinks he is getting more and more confused. He laughs when he is found to be forgetful, saying Im just getting old! After the nurse completes a thorough neurologic assessment, which findings would be indicative of Alzheimer disease? Select all that apply. a. Occasionally forgetting names or appointments b. Difficulty performing familiar tasks, such as placing a telephone call c. Misplacing items, such as putting dish soap in the refrigerator d. Sometimes having trouble finding the right word e. Rapid mood swings, from calm to tears, for no apparent reason f. Getting lost in ones own neighborhood

ANS: B, C, E, F Difficulty performing familiar tasks, misplacing items, rapid mood swings, and getting lost in ones own neighborhood can be warning signs of Alzheimer disease. Occasionally forgetting names or appointments, and sometimes having trouble finding the right word are part of normal aging.

The nurse is testing the hearing of a 78-year-old man and is reminded of the changes in hearing that occur with aging that include which of the following? (Select all that apply.) a. Hearing loss related to aging begins in the mid-40s. b. Progression of hearing loss is slow. c. The aging person has low-frequency tone loss. d. The aging person may find it harder to hear consonants than vowels. e. Sounds may be garbled and difficult to localize. f. Hearing loss reflects nerve degeneration of the middle ear.

ANS: B, D, E Presbycusis is a type of hearing loss that occurs with aging and is found in 60% of those older than 65 years of age. It is a gradual sensorineural loss caused by nerve degeneration in the inner ear or auditory nerve, and it slowly progresses after age 50 years. The person first notices a high-frequency tone loss; it is harder to hear consonants (high-pitched components of speech) than vowels, which makes words sound garbled. The ability to localize sound is also impaired.

The nurse is assessing a 1-month-old infant at his well-baby checkup. Which assessment findings are appropriate for this age? (Select all that apply.) a. Head circumference equal to chest circumference b. Head circumference greater than chest circumference c. Head circumference less than chest circumference d. Fontanelles firm and slightly concave e. Absent tonic neck reflex f. Nonpalpable cervical lymph nodes

ANS: B, D, F An infant's head circumference is larger than the chest circumference. At age 2 years, both measurements are the same. During childhood, the chest circumference grows to exceed the head circumference by 5 to 7 cm. The fontanelles should feel firm and slightly concave in the infant, and they should close by age 9 months. The tonic neck reflex is present until age 3 to 4 months, and cervical lymph nodes are normally nonpalpable in an infant.

While performing a well-child assessment on a 5-year-old, the nurse notes the presence of palpable, bilateral, cervical, and inguinal lymph nodes. They are approximately 0.5 cm in size, round, mobile, and nontender. The nurse documents that the child: a. Has chronic allergies. b. Has an infection. c. Has normal findings for a 5-year-old child. d. Should be referred for additional evaluation.

ANS: C Palpable lymph nodes are normal in children until puberty when the lymphoid tissue begins to atrophy. Lymph nodes may be up to 1 cm in size in the cervical and inguinal areas but are discrete, movable, and nontender.

During an examination, a patient states that she was diagnosed with open-angle glaucoma 2 years ago. The nurse assesses for characteristics of open-angle glaucoma. Which of these are characteristics of open-angle glaucoma? (Select all that apply.) a. Patient may experience sensitivity to light, nausea, and halos around lights. b. Patient experiences tunnel vision in the late stages. c. Immediate treatment is needed. d. Vision loss begins with peripheral vision. e. Open-angle glaucoma causes sudden attacks of increased pressure that cause blurred vision. f. Virtually no symptoms are exhibited.

ANS: B, D, F Open-angle glaucoma is the most common type of glaucoma; virtually no symptoms are exhibited. Vision loss begins with the peripheral vision, which often goes unnoticed because individuals learn to compensate intuitively by turning their heads. The other characteristics are those of closed-angle glaucoma.

The nurse notices that an infant has a large, soft lump on the side of his head and that his mother is very concerned. She tells the nurse that she noticed the lump approximately 8 hours after her baby's birth and that it seems to be getting bigger. The nurse explains that this likely is: a. Hydrocephalus. b. Craniosynostosis. c. Cephalhematoma. d. Caput succedaneum.

ANS: C A cephalhematoma is a subperiosteal hemorrhage that is the result of birth trauma. It is soft, fluctuant, and well defined over one cranial bone. It appears several hours after birth and gradually increases in size.

A patient has a severed spinal nerve as a result of trauma. Which statement is true in this situation? a. Because there are 31 pairs of spinal nerves, no effect results if only one nerve is severed. b. The dermatome served by this nerve will no longer experience any sensation. c. The adjacent spinal nerves will continue to carry sensations for the dermatome served by the severed nerve. d. A severed spinal nerve will only affect motor function of the patient because spinal nerves have no sensory component.

ANS: C A dermatome is a circumscribed skin area that is primarily supplied from one spinal cord segment through a particular spinal nerve. The dermatomes overlap, which is a form of biologic insurance; that is, if one nerve is severed, then most of the sensations can be transmitted by the spinal nerve above and the spinal nerve below the severed nerve.

A patient has been shown to have a sensorineural hearing loss. During the assessment, it would be important for the nurse to: a. Speak loudly so the patient can hear the questions. b. Assess for middle ear infection as a possible cause. c. Ask the patient about current medications. d. Look for the source of the obstruction in the external ear.

ANS: C A simple increase in amplitude may not enable the person to understand spoken words. Sensorineural hearing loss may be caused by presbycusis, which is a gradual nerve degeneration that occurs with aging and by ototoxic drugs, which affect the hair cells in the cochlea.

During an admission assessment, the nurse notices that a male patient has an enlarged and rather thick skull. The nurse suspects acromegaly and assesses the patient for: a. Exophthalmos. b. Sunken eyes. c. Coarse facial features. d. Rounded moonlike face.

ANS: C Acromegaly is excessive secretion of growth hormone that creates an enlarged skull and thickened cranial bones. Patients have elongated heads, massive faces, prominent noses and lower jaws, heavy eyebrow ridges, and coarse facial features. Exophthalmos is associated with hyperthyroidism. A round moonlike face is seen with Cushing's syndrome. Sunken eyes are reflective of a cachectic appearance. (See Table 14-4; 14-5).

During an examination of a female patient, the nurse notes lymphadenopathy and suspects an acute infection. Acutely infected lymph nodes would be: a. Clumped. b. Unilateral. c. Firm but freely movable. d. Firm and nontender.

ANS: C Acutely infected lymph nodes are bilateral, enlarged, warm, tender, and firm but freely movable. Unilaterally enlarged nodes that are firm and nontender may indicate cancer.

A patient with a middle ear infection asks the nurse, "What does the middle ear do?" The nurse responds by telling the patient that the middle ear functions to: a. Maintain balance. b. Interpret sounds as they enter the ear. c. Conduct vibrations of sounds to the inner ear. d. Increase amplitude of sound for the inner ear to function.

ANS: C Among its other functions, the middle ear conducts sound vibrations from the outer ear to the central hearing apparatus in the inner ear. The other responses are not functions of the middle ear.

The nurse is performing an assessment on a 65-year-old man. He reports a crusty nodule behind the pinna. It intermittently bleeds and has not healed over the past 6 months. On physical assessment, the nurse finds an ulcerated crusted nodule with an indurated base. The preliminary analysis in this situation is that this: a. Is most likely a benign sebaceous cyst. b. Is most likely a keloid. c. Could be a potential carcinoma, and the patient should be referred for a biopsy. d. Is a tophus, which is common in the older adult and is a sign of gout.

ANS: C An ulcerated crusted nodule with an indurated base that fails to heal is characteristic of a carcinoma. These lesions fail to heal and intermittently bleed. Individuals with such symptoms should be referred for a biopsy (see Table 16-2). The other responses are not correct.

A patient has been admitted after an accident at work. During the assessment, the patient is having trouble hearing and states, "I don't know what the matter is. All of a sudden, I can't hear you out of my left ear!" What should the nurse do next? a. Make note of this finding for the report to the next shift b. Prepare to remove cerumen from the patient's ear c. Notify the patient's health care provider d. Irrigate the ear with rubbing alcohol

ANS: C Any sudden loss of hearing in one or both ears that is not associated with an upper respiratory infection needs to be reported at once to the patient's health care provider. Hearing loss associated with trauma is often sudden. Irrigating the ear or removing cerumen is not appropriate at this time.

When taking the health history on a patient with a seizure disorder, the nurse assesses whether the patient has an aura. Which of these would be the best question for obtaining this information? a. Does your muscle tone seem tense or limp? b. After the seizure, do you spend a lot of time sleeping? c. Do you have any warning sign before your seizure starts? d. Do you experience any color change or incontinence during the seizure?

ANS: C Aura is a subjective sensation that precedes a seizure; it could be auditory, visual, or motor. The other questions do not solicit information about an aura.

A mother brings in her newborn infant for an assessment and tells the nurse that she has noticed that whenever her newborn's head is turned to the right side, she straightens out the arm and leg on the same side and flexes the opposite arm and leg. After observing this on examination, the nurse tells her that this reflex is: a. Abnormal and is called the atonic neck reflex. b. Normal and should disappear by the first year of life. c. Normal and is called the tonic neck reflex, which should disappear between 3 and 4 months of age. d. Abnormal and the baby should be flexing the arm and leg on the right side of his body when the head is turned to the right.

ANS: C By 2 weeks, the infant shows the tonic neck reflex when supine and the head is turned to one side (extension of same arm and leg, flexion of opposite arm and leg). The tonic neck reflex disappears between 3 and 4 months of age.

A man who was found wandering in a park at 2 AM has been brought to the emergency department for an examination; he said he fell and hit his head. During the examination, the nurse asks him to use his index finger to touch the nurses finger, then his own nose, then the nurses finger again (which has been moved to a different location). The patient is clumsy, unable to follow the instructions, and overshoots the mark, missing the finger. The nurse should suspect which of the following? a. Cerebral injury b. Cerebrovascular accident c. Acute alcohol intoxication d. Peripheral neuropathy

ANS: C During the finger-to-finger test, if the person has clumsy movement with overshooting the mark, either a cerebellar disorder or acute alcohol intoxication should be suspected. The persons movements should be smooth and accurate. The other options are not correct.

The nurse is preparing to perform an otoscopic examination of a newborn infant. Which statement regarding this examination is true? a. Immobility of the drum is a normal finding. b. An injected membrane would indicate an infection. c. The normal membrane may appear thick and opaque. d. The appearance of the membrane is identical to that of an adult.

ANS: C During the first few days after the birth, the tympanic membrane of a newborn often appears thickened and opaque. It may look infected and have a mild redness from increased vascularity. The other statements are not correct.

A visitor from Poland, who does not speak English, seems to be somewhat apprehensive about the nurse examining his neck. He would probably be more comfortable with the nurse examining his thyroid gland from: a. Behind with the nurse's hands placed firmly around his neck. b. The side with the nurse's eyes averted toward the ceiling and thumbs on his neck. c. The front with the nurse's thumbs placed on either side of his trachea and his head tilted forward. d. The front with the nurse's thumbs placed on either side of his trachea and his head tilted backward.

ANS: C Examining this patient's thyroid gland from the back may be unsettling for him. It would be best to examine his thyroid gland using the anterior approach, asking him to tip his head forward and to the right and then to the left.

A mother asks when her newborn infant's eyesight will be fully developed. The nurse should reply: a. "Vision is not fully developed until 2 years of age." b. "Infants develop the ability to focus on an object at approximately 8 months of age." c. "By approximately 3 months of age, infants develop more coordinated eye movements and can fixate on an object." d. "Most infants have uncoordinated eye movements for the first year of life."

ANS: C Eye movements may be poorly coordinated at birth, but by 3 to 4 months of age, the infant should establish binocularity and should be able to fixate simultaneously on a single image with both eyes.

During a well-baby checkup, the nurse notices that a 1-week-old infant's face looks small compared with his cranium, which seems enlarged. On further examination, the nurse also notices dilated scalp veins and downcast or "setting sun" eyes. The nurse suspects which condition? a. Craniotabes b. Microcephaly c. Hydrocephalus d. Caput succedaneum

ANS: C Hydrocephalus occurs with the obstruction of drainage of cerebrospinal fluid that results in excessive accumulation, increasing intracranial pressure, and an enlargement of the head. The face looks small, compared with the enlarged cranium, and dilated scalp veins and downcast or "setting sun" eyes are noted. (See Table 14-2.) Craniotabes is a softening of the skull's outer layer. Microcephaly is an abnormally small head. A caput succedaneum is edematous swelling and ecchymosis of the presenting part of the head caused by birth trauma.

A patient's thyroid gland is enlarged, and the nurse is preparing to auscultate the thyroid gland for the presence of a bruit. A bruit is a __________ sound that is heard best with the __________ of the stethoscope. a. Low gurgling; diaphragm b. Loud, whooshing, blowing; bell c. Soft, whooshing, pulsatile; bell d. High-pitched tinkling; diaphragm

ANS: C If the thyroid gland is enlarged, then the nurse should auscultate it for the presence of a bruit, which is a soft, pulsatile, whooshing, blowing sound heard best with the bell of the stethoscope.

During an assessment of a 20-year-old Asian patient, the nurse notices that he has dry, flaky cerumen in his canal. The nurse recognizes this as: a. The result of lesions from eczema in his ear. b. Poor hygiene practices. c. A normal finding and that no further follow-up is necessary. d. Indicative of change in cilia and requires further assessment.

ANS: C Individuals of Asian or Indigenous descent are more likely to have dry cerumen, whereas those of African or European descent usually have wet cerumen.

The nurse is performing a neurologic assessment on a 41-year-old woman with a history of diabetes. When testing her ability to feel the vibrations of a tuning fork, the nurse notices that the patient is unable to feel vibrations on the great toe or ankle bilaterally, but she is able to feel vibrations on both patellae. Given this information, what would the nurse suspect? a. Hyperalgesia b. Hyperesthesia c. Peripheral neuropathy d. Lesion of sensory cortex

ANS: C Loss of vibration sense occurs with peripheral neuropathy (e.g., diabetes and alcoholism). Peripheral neuropathy is worse at the feet and gradually improves as the examiner moves up the leg, as opposed to a specific nerve lesion, which has a clear zone of deficit for its dermatome. The other responses are incorrect.

A woman comes to the clinic and states, "I've been sick for so long! My eyes have gotten so puffy, and my eyebrows and hair have become coarse and dry." The nurse will assess for other signs and symptoms of: a. Cachexia. b. Parkinson's disease. c. Myxedema. d. Scleroderma.

ANS: C Myxedema (hypothyroidism) is a deficiency of the thyroid hormone that, when severe, causes a nonpitting edema or myxedema. The patient has a puffy edematous face, especially around the eyes (periorbital edema); coarse facial features; dry skin; and dry, coarse hair and eyebrows. (See Table 14-5 Thyroid Hormone Disorders.)

A 70-year-old patient tells the nurse that he has noticed that he is having trouble hearing, especially in large groups. He says that he "can't always tell where the sound is coming from" and the words often sound "mixed up." What might the nurse suspect as the cause for this change? a. Atrophy of the apocrine glands b. Cilia becoming coarse and stiff c. Nerve degeneration in the inner ear d. Scarring of the tympanic membrane

ANS: C Presbycusis is a type of hearing loss that occurs in 60% of those older than 65 years of age, even in those living in a quiet environment. This sensorineural loss is gradual and caused by nerve degeneration in the inner ear. Words sound garbled, and the ability to localize sound is also impaired. This communication dysfunction is accentuated when background noise is present.

A 60-year-old man with suspected ptosis of one eye is at the clinic for an eye examination. The nurse confirms ptosis by: a. Performing the confrontation test. b. Assessing the patient's near vision. c. Observing the distance between the palpebral fissures. d. Performing the corneal light test, and looking for symmetry of the light reflex.

ANS: C Ptosis is a drooping of the upper eyelid that would be apparent by observing the distance between the upper and lower eyelids. The confrontation test measures peripheral vision. Measuring near vision or the corneal light test does not check for ptosis.

A patient is unable to perform rapid alternating movements such as rapidly patting her knees. The nurse should document this inability as: a. Ataxia. b. Astereognosis. c. Presence of dysdiadochokinesia. d. Loss of kinesthesia.

ANS: C Slow clumsy movements and the inability to perform rapid alternating movements occur with cerebellar disease. The condition is termed dysdiadochokinesia. Ataxia is an uncoordinated or unsteady gait. Astereognosis is the inability to identify an object by feeling it. Kinesthesia is the persons ability to perceive passive movement of the extremities or the loss of position sense.

During an assessment of an 80-year-old patient, the nurse notices the following: an inability to identify vibrations at her ankle and to identify the position of her big toe, a slower and more deliberate gait, and a slightly impaired tactile sensation. All other neurologic findings are normal. The nurse should interpret that these findings indicate: a. CN dysfunction. b. Lesion in the cerebral cortex. c. Normal changes attributable to aging. d. Demyelination of nerves attributable to a lesion.

ANS: C Some aging adults show a slower response to requests, especially for those calling for coordination of movements. The findings listed are normal in the absence of other significant abnormal findings. The other responses are incorrect.

The nurse is caring for a patient who has just had neurosurgery. To assess for increased intracranial pressure, what would the nurse include in the assessment? a. CNs, motor function, and sensory function b. Deep tendon reflexes, vital signs, and coordinated movements c. Level of consciousness, motor function, pupillary response, and vital signs d. Mental status, deep tendon reflexes, sensory function, and pupillary response

ANS: C Some hospitalized persons have head trauma or a neurologic deficit from a systemic disease process. These people must be closely monitored for any improvement or deterioration in neurologic status and for any indication of increasing intracranial pressure. The nurse should use an abbreviation of the neurologic examination in the following sequence: level of consciousness, motor function, pupillary response, and vital signs.

When examining a patient's eyes, the nurse uses eye drops to stimulate the sympathetic branch of the autonomic nervous system to: a. Cause pupillary constriction. b. Adjust the eye for near vision. c. Elevate the eyelid and dilate the pupil. d. Cause contraction of the ciliary body.

ANS: C Stimulation of the sympathetic branch of the autonomic nervous system dilates the pupil and elevates the eyelid. Parasympathetic nervous system stimulation causes the pupil to constrict. The muscle fibres of the iris contract the pupil in bright light to accommodate for near vision. The ciliary body controls the thickness of the lens.

A physician tells the nurse that a patient's vertebra prominens is tender and asks the nurse to re-evaluate the area in 1 hour. The area of the body the nurse will assess is: a. Just above the diaphragm. b. Just lateral to the knee cap. c. At the level of the C7 vertebra. d. At the level of the T11 vertebra.

ANS: C The C7 vertebra has a long spinous process, called the vertebra prominens, which is palpable when the head is flexed.

While assessing a 7-month-old infant, the nurse makes a loud noise and notices the following response: abduction and flexion of the arms and legs; fanning of the fingers, and curling of the index finger and thumb in a C position, followed by the infant bringing in the arms and legs to the body. What does the nurse know about this response? a. This response could indicate brachial nerve palsy. b. This reaction is an expected startle response at this age. c. This reflex should have disappeared between 1 and 4 months of age. d. This response is normal as long as the movements are bilaterally symmetric.

ANS: C The Moro reflex is present at birth and usually disappears at 1 to 4 months. Absence of the Moro reflex in the newborn or its persistence after 5 months of age indicates severe central nervous system injury. The other responses are incorrect.

The nurse is preparing to assess the visual acuity of a 16-year-old patient. How should the nurse proceed? a. Perform the confrontation test. b. Ask the patient to read the print on a handheld Jaeger card. c. Use the Snellen chart positioned 20 feet away from the patient. d. Determine the patient's ability to read newsprint at 14 inches (35 cm) from eye.

ANS: C The Snellen alphabet chart is the most commonly used and most accurate measure of visual acuity. The confrontation test is a gross measure of peripheral vision. The Jaeger card or newspaper tests are used to test near vision.

A 30-year-old woman tells the nurse that she has been very unsteady and has had difficulty in maintaining her balance. Which area of the brain that is related to these findings would concern the nurse? a. Thalamus b. Brainstem c. Cerebellum d. Extrapyramidal tract

ANS: C The cerebellar system coordinates movement, maintains equilibrium, and helps maintain posture. The thalamus is the primary relay station where sensory pathways of the spinal cord, cerebellum, and brainstem form synapses on their way to the cerebral cortex. The brainstem consists of the midbrain, pons, and medulla and has various functions, especially concerning autonomic centers. The extrapyramidal tract maintains muscle tone for gross automatic movements, such as walking.

During an assessment, the nurse notices that an older adult patient has tears rolling down his face from his left eye. Closer examination reveals that the lower lid is loose and rolling outward. The patient complains of his eye feeling "dry and itchy." Which action by the nurse is correct? a. Assessing the eye for a possible foreign body b. Documenting the finding as ptosis c. Assessing for other signs of ectropion d. Contacting the referring physician; these are signs of basal cell carcinoma

ANS: C The condition described is known as ectropion, and it occurs in older adults and is attributable to atrophy of the elastic and fibrous tissues. The lower lid does not approximate to the eyeball, and, as a result, the puncta cannot effectively siphon tears; excessive tearing results. Ptosis is a drooping of the upper eyelid. These signs do not suggest the presence of a foreign body in the eye or basal cell carcinoma.

The nurse is examining a patient's ears and notices a yellow waxy substance in the external canal. The nurse recognizes this: a. As a sign of an ear infection. b. As indicative of poor ear hygiene. c. As protection and lubrication of the ear. d. As necessary for transmitting sound through the auditory canal.

ANS: C The ear is lined with glands that secrete cerumen, which is a yellow waxy material that lubricates and protects the ear.

A patient, an 85-year-old woman, is concerned that the bones in her face have become more noticeable. The nurse tells her that: a. Diets low in protein and high in carbohydrates may cause enhanced facial bones. b. Bones can become more noticeable if the person does not use a dermatologically approved moisturizer. c. More noticeable facial bones are probably caused by a combination of factors related to aging, such as decreased elasticity, subcutaneous fat, and moisture in her skin. d. Facial skin becomes more elastic with age. This increased elasticity causes the skin to be more taught, drawing attention to the facial bones.

ANS: C The facial bones and orbits appear more prominent in the aging adult, and the facial skin sags, which is attributable to decreased elasticity, decreased subcutaneous fat, and decreased moisture in the skin.

The nurse is reviewing the function of the cranial nerves (CNs). Which CN is responsible for conducting nerve impulses to the brain from the organ of Corti? a. I b. III c. VIII d. XI

ANS: C The nerve impulses are conducted by the auditory portion of CN VIII to the brain.

When the nurse is testing the triceps reflex, what is the expected response? a. Flexion of the hand b. Pronation of the hand c. Extension of the forearm d. Flexion of the forearm

ANS: C The normal response of the triceps reflex is extension of the forearm. The normal response of the biceps reflex causes flexion of the forearm. The other responses are incorrect.

The nurse is performing an otoscopic examination on an adult. Which of these actions is correct? a. Tilting the person's head forward during the examination b. Once the speculum is in the ear, releasing the traction c. Pulling the pinna up and back before inserting the speculum d. Using the smallest speculum to decrease the amount of discomfort

ANS: C The pinna is pulled up and back on an adult or older child, which helps straighten the "S" shape of the canal. Traction should not be released on the ear until the examination is completed and the otoscope is removed.

When a light is directed across the iris of a patient's eye from the temporal side, the nurse is assessing for: a. Drainage from dacryocystitis. b. Presence of conjunctivitis over the iris. c. Presence of shadows, which may indicate glaucoma. d. Scattered light reflex, which may be indicative of cataracts.

ANS: C The presence of shadows in the anterior chamber may be a sign of acute angle-closure glaucoma. The normal iris is flat and creates no shadows. This method is not correct for the assessment of dacryocystitis, conjunctivitis, or cataracts.

When using the ophthalmoscope to assess a patient's eyes, the nurse notices a red glow in the patient's pupils. The nurse will: a. Document that an opacity is present in the lens or cornea. b. Check the light source of the ophthalmoscope to verify that it is functioning. c. Continue with the examination knowing that the red glow is a normal finding. d. Refer the patient for further evaluation.

ANS: C The red glow filling the person's pupil is the red reflex and is a normal finding caused by the reflection of the ophthalmoscope light off the inner retina. The other responses are not correct.

A patient with a lack of oxygen to his heart will have pain in his chest and possibly in the shoulder, arms, or jaw. The nurse knows that the best explanation why this occurs is which one of these statements? a. A problem exists with the sensory cortex and its ability to discriminate the location. b. The lack of oxygen in his heart has resulted in decreased amount of oxygen to the areas experiencing the pain. c. The sensory cortex does not have the ability to localize pain in the heart; consequently, the pain is felt elsewhere. d. A lesion has developed in the dorsal root, which is preventing the sensation from being transmitted normally.

ANS: C The sensory cortex is arranged in a specific pattern, forming a corresponding map of the body. Pain in the right hand is perceived at a specific spot on the map. Some organs, such as the heart, liver, and spleen, are absent from the brain map. Pain originating in these organs is referred because no felt image exists in which to have pain. Pain is felt by proxy, that is, by another body part that does have a felt image. The other responses are not correct explanations.

The area of the nervous system that is responsible for mediating reflexes is the: a. Medulla. b. Cerebellum. c. Spinal cord. d. Cerebral cortex.

ANS: C The spinal cord is the main highway for ascending and descending fiber tracts that connect the brain to the spinal nerves; it is responsible for mediating reflexes.

While gathering equipment after an injection, a nurse accidentally received a prick from an improperly capped needle. To interpret this sensation, which of these areas must be intact? a. Corticospinal tract, medulla, and basal ganglia b. Pyramidal tract, hypothalamus, and sensory cortex c. Lateral spinothalamic tract, thalamus, and sensory cortex d. Anterior spinothalamic tract, basal ganglia, and sensory cortex

ANS: C The spinothalamic tract contains sensory fibers that transmit the sensations of pain, temperature, and crude or light touch. Fibers carrying pain and temperature sensations ascend the lateral spinothalamic tract, whereas the sensations of crude touch form the anterior spinothalamic tract. At the thalamus, the fibers synapse with another sensory neuron, which carries the message to the sensory cortex for full interpretation. The other options are not correct.

During an examination of a patient in her third trimester of pregnancy, the nurse notices that the patient's thyroid gland is slightly enlarged. No enlargement had been previously noticed. The nurse recognizes that the patient: a. Has an iodine deficiency. b. Is exhibiting early signs of goitre. c. Is exhibiting a normal enlargement of the thyroid gland during pregnancy. d. Needs further testing for possible thyroid cancer.

ANS: C The thyroid gland enlarges slightly during pregnancy because of hyperplasia of the tissue and increased vascularity.

During an examination, the patient states he is hearing a buzzing sound and says, "It is driving me crazy!" The nurse recognizes that this symptom indicates: a. Vertigo. b. Pruritus. c. Tinnitus. d. Cholesteatoma.

ANS: C Tinnitus is a sound that comes from within a person; it can be a ringing, crackling, or buzzing sound. It accompanies some hearing or ear disorders.

When assessing the pupillary light reflex, the nurse will: a. Shine a penlight from directly in front of the patient, and inspect for pupillary constriction. b. Ask the patient to follow the penlight in eight directions, and observe for bilateral pupil constriction. c. Shine a light across the pupil from the side, and observe for direct and consensual pupillary constriction. d. Ask the patient to focus on a distant object and then to follow the penlight to approximately 7 cm from the nose.

ANS: C To test the pupillary light reflex, the nurse should advance a light in from the side and note the direct and consensual pupillary constriction.

When examining the face of a patient, the nurse is aware that the two pairs of salivary glands that are accessible for examination are the ___________ and ___________ glands. a. Occipital; submental b. Parotid; jugulodigastric c. Parotid; submandibular d. Submandibular; occipital

ANS: C Two pairs of salivary glands accessible to examination on the face are the parotid glands, which are in the cheeks over the mandible, anterior to and below the ear; and the submandibular glands, which are beneath the mandible at the angle of the jaw. The parotid glands are normally nonpalpable.

In a patient who has anisocoria, the nurse would expect to observe: a. Dilated pupils. b. Excessive tearing. c. Pupils of unequal size. d. Uneven curvature of the lens.

ANS: C Unequal pupil size is termed anisocoria. It normally exists in 5% of the population but may also be indicative of central nervous system disease.

A patient who is 7 months pregnant is at the clinic for her routine checkup. During assessment the nurse notes that the patient's thyroid is palpable. The nurse will: a. Refer the patient to a thyroid specialist. b. Send the patient for laboratory tests for thyroid hormones. c. Document the findings as normal. d. Ask a colleague to check the findings.

ANS: C Usually the normal adult thyroid is not palpable. However, the thyroid gland may be palpable normally during pregnancy.

The nurse is performing an assessment on a 29-year-old woman who visits the clinic complaining of always dropping things and falling down. While testing rapid alternating movements, the nurse notices that the woman is unable to pat both of her knees. Her response is extremely slow and she frequently misses. What should the nurse suspect? a. Vestibular disease b. Lesion of CN IX c. Dysfunction of the cerebellum d. Inability to understand directions

ANS: C When a person tries to perform rapid, alternating movements, responses that are slow, clumsy, and sloppy are indicative of cerebellar disease. The other responses are incorrect.

During an examination, the nurse finds that a patient's left temporal artery is tortuous and feels hardened and tender, compared with the right temporal artery. The nurse suspects which condition? a. Crepitation b. Mastoiditis c. Temporal arteritis d. Bell's palsy

ANS: C With temporal arteritis, the artery appears more tortuous and feels hardened and tender. These assessment findings are not consistent with the other responses.

The nurse is testing superficial reflexes on an adult patient. When stroking up the lateral side of the sole and across the ball of the foot, the nurse notices the plantar flexion of the toes. How should the nurse document this finding? a. Positive Babinski sign b. Plantar reflex abnormal c. Plantar reflex present d. Plantar reflex 2+ on a scale from 0 to 4+

ANS: C With the same instrument, the nurse should draw a light stroke up the lateral side of the sole of the foot and across the ball of the foot, similar to an upside-down J. The normal response is plantar flexion of the toes and sometimes of the entire foot. A positive Babinski sign is abnormal and occurs with the response of dorsiflexion of the big toe and fanning of all toes. The plantar reflex is not graded on a 0 to 4+ scale.

While the nurse is taking the history of a 68-year-old patient who sustained a head injury 3 days earlier, he tells the nurse that he is on a cruise ship and is 30 years old. The nurse knows that this finding is indicative of a(n): a. Great sense of humor. b. Uncooperative behavior. c. Inability to understand questions. d. Decreased level of consciousness.

ANS: D A change in consciousness may be subtle. The nurse should notice any decreasing level of consciousness, disorientation, memory loss, uncooperative behavior, or even complacency in a previously combative person. The other responses are incorrect.

A patient comes into the emergency department after an accident at work. A machine blew dust into his eyes, and he was not wearing safety glasses. The nurse examines his corneas by shining a light from the side across the cornea. The nurse recognizes that the patient may have a corneal abrasion when: a. The corneas are smooth and clear. b. The lens behind the cornea is opaque. c. There are areas of bleeding across the cornea. d. There is a shattered look to the light rays reflecting off the cornea.

ANS: D A corneal abrasion causes irregular ridges in reflected light, which produce a shattered appearance to light rays. No opacities should be observed in the cornea. The other responses are not correct.

Which of these statements about the peripheral nervous system is correct? a. The CNs enter the brain through the spinal cord. b. Efferent fibers carry sensory input to the central nervous system through the spinal cord. c. The peripheral nerves are inside the central nervous system and carry impulses through their motor fibers. d. The peripheral nerves carry input to the central nervous system by afferent fibers and away from the central nervous system by efferent fibers.

ANS: D A nerve is a bundle of fibers outside of the central nervous system. The peripheral nerves carry input to the central nervous system by their sensory afferent fibers and deliver output from the central nervous system by their efferent fibers. The other responses are not related to the peripheral nervous system.

When the nurse asks a 68-year-old patient to stand with his feet together and arms at his side with his eyes closed, he starts to sway and moves his feet farther apart. The nurse would document this finding as: a. Ataxia. b. Lack of coordination. c. Negative Homans sign. d. Positive Romberg sign.

ANS: D Abnormal findings for the Romberg test include swaying, falling, and a widening base of the feet to avoid falling. A positive Romberg sign is a loss of balance that is increased by the closing of the eyes. Ataxia is an uncoordinated or unsteady gait. Homans sign is used to test the legs for deep-vein thrombosis.

A 70-year-old woman tells the nurse that every time she gets up in the morning or after shes been sitting, she gets really dizzy and feels like she is going to fall over. The nurses best response would be: a. Have you been extremely tired lately? b. You probably just need to drink more liquids. c. Ill refer you for a complete neurologic examination. d. You need to get up slowly when youve been lying down or sitting.

ANS: D Aging is accompanied by a progressive decrease in cerebral blood flow. In some people, this decrease causes dizziness and a loss of balance with a position change. These individuals need to be taught to get up slowly. The other responses are incorrect.

A 32-year-old woman tells the nurse that she has noticed very sudden, jerky movements mainly in her hands and arms. She says, They seem to come and go, primarily when I am trying to do something. I havent noticed them when Im sleeping. This description suggests: a. Tics. b. Athetosis. c. Myoclonus. d. Chorea.

ANS: D Chorea is characterized by sudden, rapid, jerky, purposeless movements that involve the limbs, trunk, or face. Chorea occurs at irregular intervals, and the movements are all accentuated by voluntary actions.

The nurse is conducting a child safety class for new mothers. Which factor places young children at risk for ear infections? a. Family history b. Air conditioning c. Excessive cerumen d. Passive cigarette smoke

ANS: D Exposure to passive and gestational smoke is a risk factor for ear infections in infants and children.

A 52-year-old patient describes the presence of occasional floaters or spots moving in front of his eyes. The nurse should: a. Examine the retina to determine the number of floaters. b. Presume the patient has glaucoma and refer patient for further testing. c. Consider these to be abnormal findings and refer patient to an ophthalmologist. d. Document the findings as common with patient age.

ANS: D Floaters are a common sensation with myopia or after middle age and are attributable to condensed vitreous fibres. Floaters or spots are not usually significant, but the acute onset of floaters may occur with retinal detachment.

An assessment of a 23-year-old patient reveals the following: an auricle that is tender and reddish-blue in colour with small vesicles. The nurse would need to know additional information that includes which of these? a. Any change in the ability to hear b. Any recent drainage from the ear c. Recent history of trauma to the ear d. Any prolonged exposure to extreme cold

ANS: D Frostbite causes reddish-blue discoloration and swelling of the auricle after exposure to extreme cold. Vesicles or bullae may develop, and the person feels pain and tenderness.

In assessing a 70-year-old patient who has had a recent cerebrovascular accident, the nurse notices right- sided weakness. What might the nurse expect to find when testing his reflexes on the right side? a. Lack of reflexes b. Normal reflexes c. Diminished reflexes d. Hyperactive reflexes

ANS: D Hyperreflexia is the exaggerated reflex observed when the monosynaptic reflex arc is released from the influence of higher cortical levels. This response occurs with upper motor neuron lesions (e.g., a cerebrovascular accident). The other responses are incorrect.

The nurse reviews causes of increased intraocular pressure (IOP) with the patient using the following explanation: a. "The pressure results from the thickness of the lens." b. "The posterior chamber increases in pressure as it accommodates increased fluid." c. "Contraction of the ciliary body in response to the aqueous within the eye increases pressure." d. "The pressure results from the amount of aqueous produced and the resistance to its outflow at the angle of the anterior chamber."

ANS: D IOP is determined by a balance between the amount of aqueous produced and the resistance to its outflow at the angle of the anterior chamber. The other responses are incorrect.

A patient is unable to read even the largest letters on the Snellen chart. What should the nurse do next? a. Refer the patient to an ophthalmologist or optometrist for further evaluation b. Assess whether the patient can count the nurse's fingers when they are placed in front of his or her eyes c. Ask the patient to put on his or her reading glasses and attempt to read the Snellen chart again d. Shorten the distance between the patient and the chart until the letters are seen and record that distance

ANS: D If the person is unable to see even the largest letters when standing 20 feet from the chart, then the nurse should shorten the distance to the chart until the letters are seen, and record that distance (e.g., "10/200"). If visual acuity is even lower, then the nurse should assess whether the person can count fingers when they are spread in front of the eyes or can distinguish light perception from a penlight. If vision is poorer than 20/30, then a referral to an ophthalmologist or optometrist is necessary, but the nurse must first assess the visual acuity.

In performing an examination of a 3-year-old child with a suspected ear infection, the nurse would: a. Omit the otoscopic examination if the child has a fever. b. Pull the ear up and back before inserting the speculum. c. Ask the mother to leave the room while examining the child. d. Perform the otoscopic examination at the end of the assessment.

ANS: D In addition to its place in the complete examination, eardrum assessment is mandatory for any infant or child requiring care for an illness or fever. For the infant or young child, the timing of the otoscopic examination is best toward the end of the complete examination.

A mother brings her 2-month-old daughter in for an examination and says, "My daughter rolled over against the wall, and now I have noticed that she has this spot that is soft on the top of her head. Is something terribly wrong?" The nurse's best response would be: a. "Perhaps that is a result of your dietary intake during pregnancy." b. "Your baby may have craniosynostosis, a disease of the sutures of the brain." c. "That 'soft spot' may be an indication of cretinism or congenital hypothyroidism." d. "That 'soft spot' is normal, and actually allows for growth of the brain during the first year of your baby's life."

ANS: D Membrane-covered "soft spots" allow for growth of the brain during the first year of life. They gradually ossify; the triangular-shaped posterior fontanelle is closed by 1 to 2 months, and the diamond-shaped anterior fontanel closes between 9 months and 2 years.

A patient is concerned that while studying for an examination he began to notice a severe headache in the left front and side of his head that was throbbing and was relieved when he lay down. He tells the nurse that his mother also had these headaches. The nurse suspects that he may be suffering from: a. Hypertension. b. Cluster headaches. c. Tension headaches. d. Migraine headaches.

ANS: D Migraine headaches tend to be supraorbital, retro-orbital, or frontotemporal with a throbbing quality. They are severe in quality and are relieved by lying down. Migraines are associated with a family history of migraine headaches.

During assessment of the lacrimal apparatus, the nurse would document the following as a normal finding: a. Presence of tears along the inner canthus b. Blocked nasolacrimal duct in a newborn infant c. Slight swelling over the upper lid and along the bony orbit if the individual has a cold d. Absence of drainage from the puncta when pressing against the inner orbital rim

ANS: D No swelling, redness, or drainage from the puncta should be observed when it is pressed. Regurgitation of fluid from the puncta, when pressed, indicates duct blockage. The lacrimal glands are not functional at birth.

The nurse is aware that the four areas in the body where lymph nodes are accessible are the: a. Head, breasts, groin, and abdomen. b. Arms, breasts, inguinal area, and legs. c. Head and neck, arms, breasts, and axillae. d. Head and neck, arms, inguinal area, and axillae.

ANS: D Nodes are located throughout the body, but they are accessible to examination only in four areas: head and neck, arms, inguinal region, and axillae.

During eye assessment of a dark-skinned patient, what normal finding does the nurse observe? a. Yellow fatty deposits over the cornea b. Pallor near the outer canthus of the lower lid c. Yellow coloration of the sclera that extends up to the iris d. Presence of small brown macules on the sclera

ANS: D Normally in dark-skinned people, small brown macules may be observed in the sclera.

Papilledema is revealed during an ophthalmic examination which the nurse recognizes as indicating: a. Retinal detachment. b. Diabetic retinopathy. c. Acute-angle glaucoma. d. Increased intracranial pressure.

ANS: D Papilledema, or choked disc, is a serious sign of increased intracranial pressure, which is caused by a space-occupying mass, such as a brain tumour or a hematoma. This pressure causes venous stasis in the globe, showing redness, congestion, and elevation of the optic disc, blurred margins, hemorrhages, and absent venous pulsations. Papilledema is not associated with the conditions in the other responses.

The mother of a 2-year-old toddler is questioning the need for tympanostomy tubes in her son's ears. The nurse will provide the following information in the teaching plan: a. The tubes will be placed in the inner ear. b. The tubes will correct sensorineural loss in children. c. The tubes do not need surgery to be inserted. d. The tubes will decrease the pressure and allow for drainage.

ANS: D Polyethylene tubes are surgically inserted into the eardrum to relieve middle ear pressure and to promote drainage of chronic or recurrent middle ear infections. Tubes spontaneously extrude in 6 months to 1 year.

The nurse is conducting an eye-screening clinic at a daycare centre. When examining a 2-year-old child, the nurse suspects that the child has a "lazy eye" and will: a. Examine the external structures of the eye. b. Assess visual acuity with the Snellen eye chart. c. Assess the child's visual fields with the confrontation test. d. Test for strabismus by performing the corneal light reflex test.

ANS: D Testing for strabismus is done by performing the corneal light reflex test and the cover test. The Snellen eye chart and confrontation test are not used to test for strabismus.

During examination of the patient's eyes, the nurse notes that the pupils become smaller when the patient looks at an object moved closer to the eyes. The nurse will document this finding as: a. Dilation of the pupils. b. Consensual light reflex. c. Conjugation. d. Accommodation.

ANS: D The accommodation reaction includes pupillary constriction and convergence of the axes of the eyes. The other responses are not correct.

A 19-year-old college student is brought to the emergency department with a severe headache he describes as, "Like nothing I've ever had before." His temperature is 40° C, and he has a stiff neck. The nurse recognizes that he needs testing for: a. A head injury. b. Cluster headaches. c. Migraine headaches. d. Meningeal inflammation.

ANS: D The acute onset of neck stiffness and pain along with headache and fever occurs with meningeal inflammation. A severe headache in an adult or child who has never had it before is a red flag. Head injury and cluster or migraine headaches are not associated with a fever or stiff neck.

A patient is admitted to the hospital with paralysis to the left side of his mouth. On assessment, the nurse notes that the patient can close his eyes but is not able to whistle or smile when asked to. The nurse recognizes that the patient needs additional assessment for: a. Cushing's syndrome. b. Parkinson's disease. c. Lower motor lesion. d. Upper motor lesion.

ANS: D With an upper motor neuron lesion that can occur with a cerebrovascular accident (CVA), the patient will have paralysis of lower facial muscles, but the upper half of the face will not be affected owing to the intact nerve from the unaffected hemisphere. The person is still able to wrinkle the forehead and close the eyes. (See Table 14-4, Abnormal Facial Appearances with Chronic Illnesses, for descriptions of the other responses.)

A 78-year-old man has a history of a cerebrovascular accident. The nurse notes that when he walks, his left arm is immobile against the body with flexion of the shoulder, elbow, wrist, and fingers and adduction of the shoulder. His left leg is stiff and extended and circumducts with each step. What type of gait disturbance is this individual experiencing? a. Scissors gait b. Cerebellar ataxia c. Parkinsonian gait d. Spastic hemiparesis

ANS: D With spastic hemiparesis, the arm is immobile against the body. Flexion of the shoulder, elbow, wrist, and fingers occurs, and adduction of the shoulder, which does not swing freely, is observed. The leg is stiff and extended and circumducts with each step. Causes of this type of gait include cerebrovascular accident.


Ensembles d'études connexes

HS 354 Sources of Retirement Income

View Set

Comptia Security+ - Chapter 1 Quiz -Mastering Security Basics

View Set

Intl Econ Exam 2- Ch 13, 14, ,15, 16, 18, 19

View Set

Chapter 29: Head and Spine injuries:

View Set